LSAT and Law School Admissions Forum

Get expert LSAT preparation and law school admissions advice from PowerScore Test Preparation.

User avatar
 Dave Killoran
PowerScore Staff
  • PowerScore Staff
  • Posts: 5850
  • Joined: Mar 25, 2011
|
#27199
Complete Question Explanation
(The complete setup for this game can be found here: lsat/viewtopic.php?t=11436)

The correct answer choice is (D)

Answer choice (A) is incorrect because L must be ranked second.

Answer choice (B) is incorrect because when both G and H are tested, H must rank better than G.

Answer choice (C) is incorrect because when M is tested both F and H must be tested.

Answer choice (D) is the correct answer choice.

Answer choice (E) is incorrect because when both F and K are tested, K must rank better than F.

Get the most out of your LSAT Prep Plus subscription.

Analyze and track your performance with our Testing and Analytics Package.